Difference between revisions of "2020 CAMO Problems/Problem 3"
(Created page with "==Problem 3== Let <math>ABC</math> be a triangle with incircle <math>\omega</math>, and let <math>\omega</math> touch <math>\overline{BC}</math>, <math>\overline{CA}</math>, <...") |
m (→Solution: author handle added) |
||
(One intermediate revision by the same user not shown) | |||
Line 3: | Line 3: | ||
==Solution== | ==Solution== | ||
− | {{ | + | Let <math>R</math> be the intersection of <math>AT</math> with <math>\omega</math> other than <math>T</math>. |
+ | |||
+ | Claim 1: <math>R</math> is the <math>D</math>-queue point of <math>DXY</math>. In particular, <math>R</math> lies on <math>(DXY)</math>. | ||
+ | Proof: First, we claim that <math>X</math> and <math>Y</math> lie on <math>TE</math> and <math>TF</math> respectively. Let <math>X_0=TE\cap DF</math> and <math>Y_0=TF\cap DE</math>. | ||
+ | By Brokard, <math>X_0Y_0</math> is the polar of <math>EF\cap DT</math> which is the line through <math>A</math> parallel to <math>BC</math>, so it coincides with the line <math>XY</math> which gives <math>X\equiv X_0</math> and <math>Y\equiv Y_0</math>. | ||
+ | The fact that <math>\measuredangle TED = \measuredangle TFD = 90^{\circ}</math> is enough to imply that <math>T</math> is the orthocenter. | ||
+ | Now, | ||
+ | \begin{align*} | ||
+ | (X, Y; A, \infty_{XY}) &\overset{T}{=} (E, F; R, T) \\ | ||
+ | &= -1 | ||
+ | \end{align*} | ||
+ | so <math>A</math> is the midpoint of <math>XY</math>, which gives the claim. | ||
+ | |||
+ | Denote <math>f_1</math> as the inversion centered at <math>T</math> swapping <math>(DXY)</math> with its nine-point circle <math>(AEF)</math>, and denote <math>f_2</math> as the inversion centered at <math>A</math> fixing <math>\omega</math>. | ||
+ | |||
+ | Claim 2: <math>R</math> lies on <math>(APQ)</math>. | ||
+ | Proof: Note that <math>f_1(Q)=P</math> which follows from the fact that <math>f_1(\omega)=XY</math>, so we are done with this claim by Power of a Point Theorem. | ||
+ | |||
+ | Finally, we return to prove that <math>R</math> is the tangency point of <math>(DXY)</math> and <math>(APQ)</math>. Inverting at <math>f_1</math> and <math>f_2</math> in that order, it suffices to show that | ||
+ | <cmath> | ||
+ | f_2(f_1(DXY))=EF | ||
+ | </cmath> | ||
+ | and | ||
+ | <cmath> | ||
+ | f_2(f_1(DXY))=H'T | ||
+ | </cmath> | ||
+ | are parallel, where <math>H'</math> is defined to be the intersection of <math>AQ</math> and <math>\omega</math> other than <math>Q</math>. | ||
+ | |||
+ | But note that <math>(Q, H'; E, F)=-1</math> and <math>Q</math> is the <math>D</math>-queue point of <math>DEF</math>, so it is well known that <math>H'</math> is the point on <math>\omega</math> such that <math>DH'\perp EF</math>. The parallelism then follows from here. | ||
+ | ~ alexanderchew | ||
==See also== | ==See also== |
Latest revision as of 05:48, 29 September 2025
Problem 3
Let be a triangle with incircle
, and let
touch
,
,
at
,
,
, respectively. Point
is the midpoint of
, and
is the point on
such that
is a diameter. Line
meets the line through
parallel to
at
and
again at
. Lines
and
intersect line
at
and
respectively. Prove that the circumcircles of
and
are tangent.
Solution
Let be the intersection of
with
other than
.
Claim 1: is the
-queue point of
. In particular,
lies on
.
Proof: First, we claim that
and
lie on
and
respectively. Let
and
.
By Brokard,
is the polar of
which is the line through
parallel to
, so it coincides with the line
which gives
and
.
The fact that
is enough to imply that
is the orthocenter.
Now,
\begin{align*}
(X, Y; A, \infty_{XY}) &\overset{T}{=} (E, F; R, T) \\
&= -1
\end{align*}
so
is the midpoint of
, which gives the claim.
Denote as the inversion centered at
swapping
with its nine-point circle
, and denote
as the inversion centered at
fixing
.
Claim 2: lies on
.
Proof: Note that
which follows from the fact that
, so we are done with this claim by Power of a Point Theorem.
Finally, we return to prove that is the tangency point of
and
. Inverting at
and
in that order, it suffices to show that
and
are parallel, where
is defined to be the intersection of
and
other than
.
But note that and
is the
-queue point of
, so it is well known that
is the point on
such that
. The parallelism then follows from here.
~ alexanderchew
See also
2020 CAMO (Problems • Resources) | ||
Preceded by Problem 2 |
Followed by Problem 4 | |
1 • 2 • 3 • 4 • 5 • 6 | ||
All CAMO Problems and Solutions |
These problems are copyrighted © by the Mathematical Association of America, as part of the American Mathematics Competitions.